Wie findet man die Green-Funktionen für die zeitabhängige inhomogene Klein-Gordon-Gleichung?

Ich versuche, die Green-Funktionen für die zeitabhängige inhomogene Klein-Gordon-Gleichung zu finden, die lautet:

[ 2 + 1 C 2 2 T 2 + κ 2 ] ψ ( R , T ) = ρ ( R , T )

In der Frage wurde erwähnt, dass ich die Funktionen des Grüns finden kann:

G R ( R , T , R ' , T ' ) = C 8 π 2 R ich D D R + e ich R C Q 2 k 2 C 2 Q 2 k 2 C 2 e ich Q ( T T ' ) D Q G A ( R , T , R ' , T ' ) = C 8 π 2 R ich D D R + e ich R C Q 2 k 2 C 2 Q 2 k 2 C 2 e ich Q ( T T ' ) D Q
‎ mit der Fourier-Transformation, aber wenn ich die Fourier-Transformation verwende, erhalte ich nicht die richtige Antwort. Die Fourier-Transformation, die ich verwende, ist diejenige, die allgemein angegeben wird als: ‎
F ( R ) = 1 2 π e ich k . R F ^ ( k ) D k
aber von dieser Transformation kann ich nichts finden G A Und G R .

Gibt es eine andere Transformation, die ich verwenden sollte, um die Green-Funktionen zu finden?

Bearbeiten Die Funktion von The Green, mit der ich ende, ist:

G A ( R , T , R ' , T ' ) = 1 ( 2 π ) 4 D 3 k D k ' 1 k 2 e ich k . ( R R ' ) e ich k ' ( T T ' )
was der hier gegebenen Antwort nicht einmal ähnlich ist!

Auf den ersten Blick sehen die erweiterten und verzögerten Funktionen ungefähr richtig aus. Was ist das Ergebnis Ihrer Berechnung? Und vielleicht kann uns das Zeigen von Arbeit helfen, das Problem zu lokalisieren.
Ich habe meine letzte Antwort auf die Frage hinzugefügt
was Sie aufgeschrieben haben, hat keine Abhängigkeit von der Masse κ , also ist es definitiv nicht richtig. Ich denke, was Sie vermissen, ist, dass Sie nach der Fourier-Transformation in der Raumzeit versuchen sollten, die Gleichung zu lösen
( | k | 2 τ 2 C 2 + κ 2 ) ψ ^ ( k , τ ) = 0
Es scheint mir, dass Sie einfach vergessen haben, dass der Massenbegriff berücksichtigt werden sollte. Außerdem: Seien Sie vorsichtig mit Ihrem κ Und k ! Das sind zwei verschiedene Buchstaben!

Antworten (1)

Ich werde versuchen, auf Dinge hinzuweisen, die meiner Meinung nach nicht in Ordnung sind.

  • Du machst Verwirrung zwischen k Und κ .
  • Es gibt kein Problem mit Ihrer Konvention für die Fourier-Transformation. Es ist nur ein wenig unkonventionell bei dieser Art von Problem (basierend auf den Büchern, die ich verwendet habe)
  • Sie müssen sich integrieren! Beachten Sie, dass das Integral, das Sie gefunden haben, hat D 3 k D k ' du musst es lösen. Der gemeinsame Weg ist, zuerst zu integrieren k ' , setze dann das Integral in Kugelkoordinaten und löse nach den Winkeln auf. Löse zum Schluss das radiale Integral.

Hast du erstmal versucht das zu lösen κ = 0 Fall? Wenn nein, sollten Sie das tun.

Ich werde hier nicht wiedergeben, was in einigen Büchern gesagt wird, stattdessen lasse ich Sie mit einigen Quellen:

  • Ich denke, die bessere Referenz ist das Hassani-Buch über mathematische Physik (Abschnitt 22.4.4), dort sind die Dinge klarer. Das Problem ist, dass er das nicht löst κ 0 Fall.
  • Eine weitere großartige Referenz (und die Ihr Problem löst) ist Eleftherios Economous Buch Green's Function on Quantum Mechanics Sec.2.2 (siehe Seite 31, Gl. 2.63 für Ihren speziellen Fall).
  • Andere Referenzen sind Bogoliubov Shirkov &15 (was meiner Meinung nach kanonisch, aber etwas verwirrend ist) und
  • Morse Feshbach Kapitel 7.
sehr nützlich. Tnx